Does any basis belong to a set of eigenfunctions of some observable?











up vote
8
down vote

favorite
2












In the Quantum Mechanics courses in ocw.mit.edu
it is said that any wave function can be expanded as a superposition of the eigenfunctions of any observable, i.e the eigenfunctions form a eigenbasis for our space.



However, if we take an arbitrary basis for our space, is that space an eigenbasis corresponding to an observable? In other words, is any basis a set of eigenfunctions of some observable?










share|cite|improve this question
























  • Related: physics.stackexchange.com/q/373357/2451
    – Qmechanic
    yesterday















up vote
8
down vote

favorite
2












In the Quantum Mechanics courses in ocw.mit.edu
it is said that any wave function can be expanded as a superposition of the eigenfunctions of any observable, i.e the eigenfunctions form a eigenbasis for our space.



However, if we take an arbitrary basis for our space, is that space an eigenbasis corresponding to an observable? In other words, is any basis a set of eigenfunctions of some observable?










share|cite|improve this question
























  • Related: physics.stackexchange.com/q/373357/2451
    – Qmechanic
    yesterday













up vote
8
down vote

favorite
2









up vote
8
down vote

favorite
2






2





In the Quantum Mechanics courses in ocw.mit.edu
it is said that any wave function can be expanded as a superposition of the eigenfunctions of any observable, i.e the eigenfunctions form a eigenbasis for our space.



However, if we take an arbitrary basis for our space, is that space an eigenbasis corresponding to an observable? In other words, is any basis a set of eigenfunctions of some observable?










share|cite|improve this question















In the Quantum Mechanics courses in ocw.mit.edu
it is said that any wave function can be expanded as a superposition of the eigenfunctions of any observable, i.e the eigenfunctions form a eigenbasis for our space.



However, if we take an arbitrary basis for our space, is that space an eigenbasis corresponding to an observable? In other words, is any basis a set of eigenfunctions of some observable?







quantum-mechanics operators hilbert-space observables






share|cite|improve this question















share|cite|improve this question













share|cite|improve this question




share|cite|improve this question








edited yesterday









Ruslan

7,85342864




7,85342864










asked yesterday









onurcanbektas

371119




371119












  • Related: physics.stackexchange.com/q/373357/2451
    – Qmechanic
    yesterday


















  • Related: physics.stackexchange.com/q/373357/2451
    – Qmechanic
    yesterday
















Related: physics.stackexchange.com/q/373357/2451
– Qmechanic
yesterday




Related: physics.stackexchange.com/q/373357/2451
– Qmechanic
yesterday










3 Answers
3






active

oldest

votes

















up vote
12
down vote



accepted











is any basis a set of eigenfunctions of some observable?




This depends on what exactly you want to count as an 'observable' and what you want to rule out.





The first example you need to consider is simply the identity operator $mathbb I$, for which any nonzero vector is an eigenvector with eigenvalue $1$. (Physically, this corresponds to the number $1$ $-$ so, in the same sense that observables in classical mechanics are functions $f(q,p)$ of position and momentum, the identity operator corresponds to the function $f(q,p) equiv 1$.) So the answer here is yes, though that might require what some people consider to be a 'trivial' observable.





In response to that, the next tempting step is to alter the request to




is any basis a set of eigenfunctions of some non-constant observable?




which rules out the identity operator and its multiples.



Once you do that, the answer is no. The way you've phrased it, the basis set need not be orthogonal, and if the basis is non-orthogonal then it's not possible to build a hermitian operator with that basis as its eigenfunctions, since any eigenbasis of a hermitian operator must be orthogonal.



(That's not quite true, as it happens, because you can have non-orthogonal basis vectors inside any degenerate eigenspace, but that returns us to the case above - the observable just acts like the identity within that subspace. In terms of the question, it means that if the basis $beta$ is non-orthogonal but it does contain two subsets $beta_1={u_i}$ and $beta_2={v_j}$ which are mutually orthogonal, then you can build a non-constant observable $A$ such that $Au_i=u : u_i$ and $A v_j = v:v_j$ for two fixed numbers $u$ and $v$. Again, though, that's basically a trivial cop-out, and it doesn't bring anything new that the identity operator didn't already.)





So, v2 of the statement needs to be fixed:




is any orthogonal basis a set of eigenfunctions of some non-constant observable?




Here the answer is yes: let $beta = { v_n : n=1,2,ldots }$ be a countable orthonormal basis. (And yes, it needs to be countable if you want any shot at a workable solution.) Then there exists a unique linear self-adjoint operator $A$ which extends
$$
A v_n = frac1n v_n
$$

to the entire Hilbert space, and that operator has $beta$ as its unique eigenbasis with no degeneracies anywhere.





However, the solution above will produce a self-adjoint operator which (although it is perfectly well-defined mathematically) can be completely inaccessible to any imaginable physical experiment on your system, so it can again be seen as not particularly satisfactory. The solution to that is to introduce the concept of a physically-accessible observable, which you construct by starting with some core set (say, $hat x$ and $hat p$) and then allowing any linear combinations, powers of existing operators, and commutators of existing operators. (This represents a shift in thinking, which is well exemplified in the shift between §2.3.1.b and §3.2.1.a of arXiv:1211.5627.)



So, with that, you may ask




is any orthogonal basis a set of eigenfunctions of some non-constant physically-accessible observable?




and here the answer is no, there's some perfectly-reasonable systems with perfectly-reasonable state spaces and perfectly-reasonable algebras of observables, which allow for perfectly-reasonable bases that are not eigenbases of any observable in the chosen algebra. (Unfortunately, though, I don't have any concrete examples at hand. But within that narrow definition, it makes for a good follow-up question if you want to post it.)





So, what's the answer? Yes. And no. Depending on what you mean by "basis" and by "observable".






share|cite|improve this answer





















  • Thanks a lot for this extensive answer.
    – onurcanbektas
    yesterday










  • Very nice answer, Emilio. I'm curious about something though: in the second case you discuss, you present a basis set $beta$, and you state that the cardinality of the set must be countable to have a workable solution. I know that in principle we can construct vector spaces with uncountable bases, mathematically there's no problem there, but is your point that such an uncountable basis is impenetrable to the tools of a theoretical physicist?
    – N. Steinle
    yesterday






  • 1




    @N.Steinle No. But if you have an uncountable basis, then either (i) your Hilbert space is not separable, in which case you're way off the beaten track of what QM usually handles, and you're not actually doing anything that's physically applicable at all, or (ii) your continuous "basis" is something like the momentum states in the 'canonical' QM, and the "basis states" don't actually live in the Hilbert space (and you'd need a rigged Hilbert space, a.k.a. a Gelfand triplet, to describe them rigorously) and you're a ways off from what the mathematical sophistication shown by OP can handle.
    – Emilio Pisanty
    yesterday






  • 1




    (while, at the same time, not actually changing the content of the answer at all - the results are exactly the same, it's just more hassle to prove them correctly.)
    – Emilio Pisanty
    yesterday










  • @EmilioPisanty Thank you for clarifying.
    – N. Steinle
    yesterday


















up vote
5
down vote














In other words, is any basis a set of eigenfunctions of some observable?




IN general, no, since we require that physical observables have certain properties. Mathematically speaking, however, a function can be expressed in any orthogonal basis, but that doesnt mean that the basis is physically meaningful.



A wavefunction may be written as a linear combination of any well defined basis (that is, it is orthonormal), and of course in principle one can define an operator that corresponds to that basis. However, this operator does not necessarily correspond to a physical observable. For instance, in the example of the harmonic oscillator potential, we may express the wavefunction of the system as a superposition of coherent states, but the operator you'd define from the basis of coherent states (a raising or lowering operator) does not represent a physical observable.



So, formally speaking, we require that physical observables be Hermitian, so that their corresponding basis is orthonormal over some interval and their eigenvalues are real-valued (so they provide real-valued expectation values). This is an axiom of the theory of quantum mechanics.






share|cite|improve this answer




























    up vote
    1
    down vote













    Probably not, but it may depend on what you mean by an observable.



    Any hermitian operator will have a complete basis set of eigenfunctions. Some hermitian operators have been found to be associated with observables. Operators representing observables are required to be hermitian because they must have real expectation values.



    Any linear sum of hermitian operators would also be hermitian, and if made up of observables that quantity could also be observed. I am unsure how to prove that the set of possible complete bases states is larger than the set that you could construct from arbitrary linear sums of existing observables, but you probably can prove that.






    share|cite|improve this answer








    New contributor




    S.D.A. Thomas is a new contributor to this site. Take care in asking for clarification, commenting, and answering.
    Check out our Code of Conduct.














    • 1




      The OP's question is essentially does every basis correspond to an observable. If you say that there exist Hermetian operators with orthonormal eigenbases that don't correspond to observables, then isn't the answer to the question just "no"?
      – Aaron Stevens
      yesterday











    Your Answer





    StackExchange.ifUsing("editor", function () {
    return StackExchange.using("mathjaxEditing", function () {
    StackExchange.MarkdownEditor.creationCallbacks.add(function (editor, postfix) {
    StackExchange.mathjaxEditing.prepareWmdForMathJax(editor, postfix, [["$", "$"], ["\\(","\\)"]]);
    });
    });
    }, "mathjax-editing");

    StackExchange.ready(function() {
    var channelOptions = {
    tags: "".split(" "),
    id: "151"
    };
    initTagRenderer("".split(" "), "".split(" "), channelOptions);

    StackExchange.using("externalEditor", function() {
    // Have to fire editor after snippets, if snippets enabled
    if (StackExchange.settings.snippets.snippetsEnabled) {
    StackExchange.using("snippets", function() {
    createEditor();
    });
    }
    else {
    createEditor();
    }
    });

    function createEditor() {
    StackExchange.prepareEditor({
    heartbeatType: 'answer',
    convertImagesToLinks: false,
    noModals: true,
    showLowRepImageUploadWarning: true,
    reputationToPostImages: null,
    bindNavPrevention: true,
    postfix: "",
    imageUploader: {
    brandingHtml: "Powered by u003ca class="icon-imgur-white" href="https://imgur.com/"u003eu003c/au003e",
    contentPolicyHtml: "User contributions licensed under u003ca href="https://creativecommons.org/licenses/by-sa/3.0/"u003ecc by-sa 3.0 with attribution requiredu003c/au003e u003ca href="https://stackoverflow.com/legal/content-policy"u003e(content policy)u003c/au003e",
    allowUrls: true
    },
    noCode: true, onDemand: true,
    discardSelector: ".discard-answer"
    ,immediatelyShowMarkdownHelp:true
    });


    }
    });














     

    draft saved


    draft discarded


















    StackExchange.ready(
    function () {
    StackExchange.openid.initPostLogin('.new-post-login', 'https%3a%2f%2fphysics.stackexchange.com%2fquestions%2f442583%2fdoes-any-basis-belong-to-a-set-of-eigenfunctions-of-some-observable%23new-answer', 'question_page');
    }
    );

    Post as a guest















    Required, but never shown

























    3 Answers
    3






    active

    oldest

    votes








    3 Answers
    3






    active

    oldest

    votes









    active

    oldest

    votes






    active

    oldest

    votes








    up vote
    12
    down vote



    accepted











    is any basis a set of eigenfunctions of some observable?




    This depends on what exactly you want to count as an 'observable' and what you want to rule out.





    The first example you need to consider is simply the identity operator $mathbb I$, for which any nonzero vector is an eigenvector with eigenvalue $1$. (Physically, this corresponds to the number $1$ $-$ so, in the same sense that observables in classical mechanics are functions $f(q,p)$ of position and momentum, the identity operator corresponds to the function $f(q,p) equiv 1$.) So the answer here is yes, though that might require what some people consider to be a 'trivial' observable.





    In response to that, the next tempting step is to alter the request to




    is any basis a set of eigenfunctions of some non-constant observable?




    which rules out the identity operator and its multiples.



    Once you do that, the answer is no. The way you've phrased it, the basis set need not be orthogonal, and if the basis is non-orthogonal then it's not possible to build a hermitian operator with that basis as its eigenfunctions, since any eigenbasis of a hermitian operator must be orthogonal.



    (That's not quite true, as it happens, because you can have non-orthogonal basis vectors inside any degenerate eigenspace, but that returns us to the case above - the observable just acts like the identity within that subspace. In terms of the question, it means that if the basis $beta$ is non-orthogonal but it does contain two subsets $beta_1={u_i}$ and $beta_2={v_j}$ which are mutually orthogonal, then you can build a non-constant observable $A$ such that $Au_i=u : u_i$ and $A v_j = v:v_j$ for two fixed numbers $u$ and $v$. Again, though, that's basically a trivial cop-out, and it doesn't bring anything new that the identity operator didn't already.)





    So, v2 of the statement needs to be fixed:




    is any orthogonal basis a set of eigenfunctions of some non-constant observable?




    Here the answer is yes: let $beta = { v_n : n=1,2,ldots }$ be a countable orthonormal basis. (And yes, it needs to be countable if you want any shot at a workable solution.) Then there exists a unique linear self-adjoint operator $A$ which extends
    $$
    A v_n = frac1n v_n
    $$

    to the entire Hilbert space, and that operator has $beta$ as its unique eigenbasis with no degeneracies anywhere.





    However, the solution above will produce a self-adjoint operator which (although it is perfectly well-defined mathematically) can be completely inaccessible to any imaginable physical experiment on your system, so it can again be seen as not particularly satisfactory. The solution to that is to introduce the concept of a physically-accessible observable, which you construct by starting with some core set (say, $hat x$ and $hat p$) and then allowing any linear combinations, powers of existing operators, and commutators of existing operators. (This represents a shift in thinking, which is well exemplified in the shift between §2.3.1.b and §3.2.1.a of arXiv:1211.5627.)



    So, with that, you may ask




    is any orthogonal basis a set of eigenfunctions of some non-constant physically-accessible observable?




    and here the answer is no, there's some perfectly-reasonable systems with perfectly-reasonable state spaces and perfectly-reasonable algebras of observables, which allow for perfectly-reasonable bases that are not eigenbases of any observable in the chosen algebra. (Unfortunately, though, I don't have any concrete examples at hand. But within that narrow definition, it makes for a good follow-up question if you want to post it.)





    So, what's the answer? Yes. And no. Depending on what you mean by "basis" and by "observable".






    share|cite|improve this answer





















    • Thanks a lot for this extensive answer.
      – onurcanbektas
      yesterday










    • Very nice answer, Emilio. I'm curious about something though: in the second case you discuss, you present a basis set $beta$, and you state that the cardinality of the set must be countable to have a workable solution. I know that in principle we can construct vector spaces with uncountable bases, mathematically there's no problem there, but is your point that such an uncountable basis is impenetrable to the tools of a theoretical physicist?
      – N. Steinle
      yesterday






    • 1




      @N.Steinle No. But if you have an uncountable basis, then either (i) your Hilbert space is not separable, in which case you're way off the beaten track of what QM usually handles, and you're not actually doing anything that's physically applicable at all, or (ii) your continuous "basis" is something like the momentum states in the 'canonical' QM, and the "basis states" don't actually live in the Hilbert space (and you'd need a rigged Hilbert space, a.k.a. a Gelfand triplet, to describe them rigorously) and you're a ways off from what the mathematical sophistication shown by OP can handle.
      – Emilio Pisanty
      yesterday






    • 1




      (while, at the same time, not actually changing the content of the answer at all - the results are exactly the same, it's just more hassle to prove them correctly.)
      – Emilio Pisanty
      yesterday










    • @EmilioPisanty Thank you for clarifying.
      – N. Steinle
      yesterday















    up vote
    12
    down vote



    accepted











    is any basis a set of eigenfunctions of some observable?




    This depends on what exactly you want to count as an 'observable' and what you want to rule out.





    The first example you need to consider is simply the identity operator $mathbb I$, for which any nonzero vector is an eigenvector with eigenvalue $1$. (Physically, this corresponds to the number $1$ $-$ so, in the same sense that observables in classical mechanics are functions $f(q,p)$ of position and momentum, the identity operator corresponds to the function $f(q,p) equiv 1$.) So the answer here is yes, though that might require what some people consider to be a 'trivial' observable.





    In response to that, the next tempting step is to alter the request to




    is any basis a set of eigenfunctions of some non-constant observable?




    which rules out the identity operator and its multiples.



    Once you do that, the answer is no. The way you've phrased it, the basis set need not be orthogonal, and if the basis is non-orthogonal then it's not possible to build a hermitian operator with that basis as its eigenfunctions, since any eigenbasis of a hermitian operator must be orthogonal.



    (That's not quite true, as it happens, because you can have non-orthogonal basis vectors inside any degenerate eigenspace, but that returns us to the case above - the observable just acts like the identity within that subspace. In terms of the question, it means that if the basis $beta$ is non-orthogonal but it does contain two subsets $beta_1={u_i}$ and $beta_2={v_j}$ which are mutually orthogonal, then you can build a non-constant observable $A$ such that $Au_i=u : u_i$ and $A v_j = v:v_j$ for two fixed numbers $u$ and $v$. Again, though, that's basically a trivial cop-out, and it doesn't bring anything new that the identity operator didn't already.)





    So, v2 of the statement needs to be fixed:




    is any orthogonal basis a set of eigenfunctions of some non-constant observable?




    Here the answer is yes: let $beta = { v_n : n=1,2,ldots }$ be a countable orthonormal basis. (And yes, it needs to be countable if you want any shot at a workable solution.) Then there exists a unique linear self-adjoint operator $A$ which extends
    $$
    A v_n = frac1n v_n
    $$

    to the entire Hilbert space, and that operator has $beta$ as its unique eigenbasis with no degeneracies anywhere.





    However, the solution above will produce a self-adjoint operator which (although it is perfectly well-defined mathematically) can be completely inaccessible to any imaginable physical experiment on your system, so it can again be seen as not particularly satisfactory. The solution to that is to introduce the concept of a physically-accessible observable, which you construct by starting with some core set (say, $hat x$ and $hat p$) and then allowing any linear combinations, powers of existing operators, and commutators of existing operators. (This represents a shift in thinking, which is well exemplified in the shift between §2.3.1.b and §3.2.1.a of arXiv:1211.5627.)



    So, with that, you may ask




    is any orthogonal basis a set of eigenfunctions of some non-constant physically-accessible observable?




    and here the answer is no, there's some perfectly-reasonable systems with perfectly-reasonable state spaces and perfectly-reasonable algebras of observables, which allow for perfectly-reasonable bases that are not eigenbases of any observable in the chosen algebra. (Unfortunately, though, I don't have any concrete examples at hand. But within that narrow definition, it makes for a good follow-up question if you want to post it.)





    So, what's the answer? Yes. And no. Depending on what you mean by "basis" and by "observable".






    share|cite|improve this answer





















    • Thanks a lot for this extensive answer.
      – onurcanbektas
      yesterday










    • Very nice answer, Emilio. I'm curious about something though: in the second case you discuss, you present a basis set $beta$, and you state that the cardinality of the set must be countable to have a workable solution. I know that in principle we can construct vector spaces with uncountable bases, mathematically there's no problem there, but is your point that such an uncountable basis is impenetrable to the tools of a theoretical physicist?
      – N. Steinle
      yesterday






    • 1




      @N.Steinle No. But if you have an uncountable basis, then either (i) your Hilbert space is not separable, in which case you're way off the beaten track of what QM usually handles, and you're not actually doing anything that's physically applicable at all, or (ii) your continuous "basis" is something like the momentum states in the 'canonical' QM, and the "basis states" don't actually live in the Hilbert space (and you'd need a rigged Hilbert space, a.k.a. a Gelfand triplet, to describe them rigorously) and you're a ways off from what the mathematical sophistication shown by OP can handle.
      – Emilio Pisanty
      yesterday






    • 1




      (while, at the same time, not actually changing the content of the answer at all - the results are exactly the same, it's just more hassle to prove them correctly.)
      – Emilio Pisanty
      yesterday










    • @EmilioPisanty Thank you for clarifying.
      – N. Steinle
      yesterday













    up vote
    12
    down vote



    accepted







    up vote
    12
    down vote



    accepted







    is any basis a set of eigenfunctions of some observable?




    This depends on what exactly you want to count as an 'observable' and what you want to rule out.





    The first example you need to consider is simply the identity operator $mathbb I$, for which any nonzero vector is an eigenvector with eigenvalue $1$. (Physically, this corresponds to the number $1$ $-$ so, in the same sense that observables in classical mechanics are functions $f(q,p)$ of position and momentum, the identity operator corresponds to the function $f(q,p) equiv 1$.) So the answer here is yes, though that might require what some people consider to be a 'trivial' observable.





    In response to that, the next tempting step is to alter the request to




    is any basis a set of eigenfunctions of some non-constant observable?




    which rules out the identity operator and its multiples.



    Once you do that, the answer is no. The way you've phrased it, the basis set need not be orthogonal, and if the basis is non-orthogonal then it's not possible to build a hermitian operator with that basis as its eigenfunctions, since any eigenbasis of a hermitian operator must be orthogonal.



    (That's not quite true, as it happens, because you can have non-orthogonal basis vectors inside any degenerate eigenspace, but that returns us to the case above - the observable just acts like the identity within that subspace. In terms of the question, it means that if the basis $beta$ is non-orthogonal but it does contain two subsets $beta_1={u_i}$ and $beta_2={v_j}$ which are mutually orthogonal, then you can build a non-constant observable $A$ such that $Au_i=u : u_i$ and $A v_j = v:v_j$ for two fixed numbers $u$ and $v$. Again, though, that's basically a trivial cop-out, and it doesn't bring anything new that the identity operator didn't already.)





    So, v2 of the statement needs to be fixed:




    is any orthogonal basis a set of eigenfunctions of some non-constant observable?




    Here the answer is yes: let $beta = { v_n : n=1,2,ldots }$ be a countable orthonormal basis. (And yes, it needs to be countable if you want any shot at a workable solution.) Then there exists a unique linear self-adjoint operator $A$ which extends
    $$
    A v_n = frac1n v_n
    $$

    to the entire Hilbert space, and that operator has $beta$ as its unique eigenbasis with no degeneracies anywhere.





    However, the solution above will produce a self-adjoint operator which (although it is perfectly well-defined mathematically) can be completely inaccessible to any imaginable physical experiment on your system, so it can again be seen as not particularly satisfactory. The solution to that is to introduce the concept of a physically-accessible observable, which you construct by starting with some core set (say, $hat x$ and $hat p$) and then allowing any linear combinations, powers of existing operators, and commutators of existing operators. (This represents a shift in thinking, which is well exemplified in the shift between §2.3.1.b and §3.2.1.a of arXiv:1211.5627.)



    So, with that, you may ask




    is any orthogonal basis a set of eigenfunctions of some non-constant physically-accessible observable?




    and here the answer is no, there's some perfectly-reasonable systems with perfectly-reasonable state spaces and perfectly-reasonable algebras of observables, which allow for perfectly-reasonable bases that are not eigenbases of any observable in the chosen algebra. (Unfortunately, though, I don't have any concrete examples at hand. But within that narrow definition, it makes for a good follow-up question if you want to post it.)





    So, what's the answer? Yes. And no. Depending on what you mean by "basis" and by "observable".






    share|cite|improve this answer













    is any basis a set of eigenfunctions of some observable?




    This depends on what exactly you want to count as an 'observable' and what you want to rule out.





    The first example you need to consider is simply the identity operator $mathbb I$, for which any nonzero vector is an eigenvector with eigenvalue $1$. (Physically, this corresponds to the number $1$ $-$ so, in the same sense that observables in classical mechanics are functions $f(q,p)$ of position and momentum, the identity operator corresponds to the function $f(q,p) equiv 1$.) So the answer here is yes, though that might require what some people consider to be a 'trivial' observable.





    In response to that, the next tempting step is to alter the request to




    is any basis a set of eigenfunctions of some non-constant observable?




    which rules out the identity operator and its multiples.



    Once you do that, the answer is no. The way you've phrased it, the basis set need not be orthogonal, and if the basis is non-orthogonal then it's not possible to build a hermitian operator with that basis as its eigenfunctions, since any eigenbasis of a hermitian operator must be orthogonal.



    (That's not quite true, as it happens, because you can have non-orthogonal basis vectors inside any degenerate eigenspace, but that returns us to the case above - the observable just acts like the identity within that subspace. In terms of the question, it means that if the basis $beta$ is non-orthogonal but it does contain two subsets $beta_1={u_i}$ and $beta_2={v_j}$ which are mutually orthogonal, then you can build a non-constant observable $A$ such that $Au_i=u : u_i$ and $A v_j = v:v_j$ for two fixed numbers $u$ and $v$. Again, though, that's basically a trivial cop-out, and it doesn't bring anything new that the identity operator didn't already.)





    So, v2 of the statement needs to be fixed:




    is any orthogonal basis a set of eigenfunctions of some non-constant observable?




    Here the answer is yes: let $beta = { v_n : n=1,2,ldots }$ be a countable orthonormal basis. (And yes, it needs to be countable if you want any shot at a workable solution.) Then there exists a unique linear self-adjoint operator $A$ which extends
    $$
    A v_n = frac1n v_n
    $$

    to the entire Hilbert space, and that operator has $beta$ as its unique eigenbasis with no degeneracies anywhere.





    However, the solution above will produce a self-adjoint operator which (although it is perfectly well-defined mathematically) can be completely inaccessible to any imaginable physical experiment on your system, so it can again be seen as not particularly satisfactory. The solution to that is to introduce the concept of a physically-accessible observable, which you construct by starting with some core set (say, $hat x$ and $hat p$) and then allowing any linear combinations, powers of existing operators, and commutators of existing operators. (This represents a shift in thinking, which is well exemplified in the shift between §2.3.1.b and §3.2.1.a of arXiv:1211.5627.)



    So, with that, you may ask




    is any orthogonal basis a set of eigenfunctions of some non-constant physically-accessible observable?




    and here the answer is no, there's some perfectly-reasonable systems with perfectly-reasonable state spaces and perfectly-reasonable algebras of observables, which allow for perfectly-reasonable bases that are not eigenbases of any observable in the chosen algebra. (Unfortunately, though, I don't have any concrete examples at hand. But within that narrow definition, it makes for a good follow-up question if you want to post it.)





    So, what's the answer? Yes. And no. Depending on what you mean by "basis" and by "observable".







    share|cite|improve this answer












    share|cite|improve this answer



    share|cite|improve this answer










    answered yesterday









    Emilio Pisanty

    80.9k21193397




    80.9k21193397












    • Thanks a lot for this extensive answer.
      – onurcanbektas
      yesterday










    • Very nice answer, Emilio. I'm curious about something though: in the second case you discuss, you present a basis set $beta$, and you state that the cardinality of the set must be countable to have a workable solution. I know that in principle we can construct vector spaces with uncountable bases, mathematically there's no problem there, but is your point that such an uncountable basis is impenetrable to the tools of a theoretical physicist?
      – N. Steinle
      yesterday






    • 1




      @N.Steinle No. But if you have an uncountable basis, then either (i) your Hilbert space is not separable, in which case you're way off the beaten track of what QM usually handles, and you're not actually doing anything that's physically applicable at all, or (ii) your continuous "basis" is something like the momentum states in the 'canonical' QM, and the "basis states" don't actually live in the Hilbert space (and you'd need a rigged Hilbert space, a.k.a. a Gelfand triplet, to describe them rigorously) and you're a ways off from what the mathematical sophistication shown by OP can handle.
      – Emilio Pisanty
      yesterday






    • 1




      (while, at the same time, not actually changing the content of the answer at all - the results are exactly the same, it's just more hassle to prove them correctly.)
      – Emilio Pisanty
      yesterday










    • @EmilioPisanty Thank you for clarifying.
      – N. Steinle
      yesterday


















    • Thanks a lot for this extensive answer.
      – onurcanbektas
      yesterday










    • Very nice answer, Emilio. I'm curious about something though: in the second case you discuss, you present a basis set $beta$, and you state that the cardinality of the set must be countable to have a workable solution. I know that in principle we can construct vector spaces with uncountable bases, mathematically there's no problem there, but is your point that such an uncountable basis is impenetrable to the tools of a theoretical physicist?
      – N. Steinle
      yesterday






    • 1




      @N.Steinle No. But if you have an uncountable basis, then either (i) your Hilbert space is not separable, in which case you're way off the beaten track of what QM usually handles, and you're not actually doing anything that's physically applicable at all, or (ii) your continuous "basis" is something like the momentum states in the 'canonical' QM, and the "basis states" don't actually live in the Hilbert space (and you'd need a rigged Hilbert space, a.k.a. a Gelfand triplet, to describe them rigorously) and you're a ways off from what the mathematical sophistication shown by OP can handle.
      – Emilio Pisanty
      yesterday






    • 1




      (while, at the same time, not actually changing the content of the answer at all - the results are exactly the same, it's just more hassle to prove them correctly.)
      – Emilio Pisanty
      yesterday










    • @EmilioPisanty Thank you for clarifying.
      – N. Steinle
      yesterday
















    Thanks a lot for this extensive answer.
    – onurcanbektas
    yesterday




    Thanks a lot for this extensive answer.
    – onurcanbektas
    yesterday












    Very nice answer, Emilio. I'm curious about something though: in the second case you discuss, you present a basis set $beta$, and you state that the cardinality of the set must be countable to have a workable solution. I know that in principle we can construct vector spaces with uncountable bases, mathematically there's no problem there, but is your point that such an uncountable basis is impenetrable to the tools of a theoretical physicist?
    – N. Steinle
    yesterday




    Very nice answer, Emilio. I'm curious about something though: in the second case you discuss, you present a basis set $beta$, and you state that the cardinality of the set must be countable to have a workable solution. I know that in principle we can construct vector spaces with uncountable bases, mathematically there's no problem there, but is your point that such an uncountable basis is impenetrable to the tools of a theoretical physicist?
    – N. Steinle
    yesterday




    1




    1




    @N.Steinle No. But if you have an uncountable basis, then either (i) your Hilbert space is not separable, in which case you're way off the beaten track of what QM usually handles, and you're not actually doing anything that's physically applicable at all, or (ii) your continuous "basis" is something like the momentum states in the 'canonical' QM, and the "basis states" don't actually live in the Hilbert space (and you'd need a rigged Hilbert space, a.k.a. a Gelfand triplet, to describe them rigorously) and you're a ways off from what the mathematical sophistication shown by OP can handle.
    – Emilio Pisanty
    yesterday




    @N.Steinle No. But if you have an uncountable basis, then either (i) your Hilbert space is not separable, in which case you're way off the beaten track of what QM usually handles, and you're not actually doing anything that's physically applicable at all, or (ii) your continuous "basis" is something like the momentum states in the 'canonical' QM, and the "basis states" don't actually live in the Hilbert space (and you'd need a rigged Hilbert space, a.k.a. a Gelfand triplet, to describe them rigorously) and you're a ways off from what the mathematical sophistication shown by OP can handle.
    – Emilio Pisanty
    yesterday




    1




    1




    (while, at the same time, not actually changing the content of the answer at all - the results are exactly the same, it's just more hassle to prove them correctly.)
    – Emilio Pisanty
    yesterday




    (while, at the same time, not actually changing the content of the answer at all - the results are exactly the same, it's just more hassle to prove them correctly.)
    – Emilio Pisanty
    yesterday












    @EmilioPisanty Thank you for clarifying.
    – N. Steinle
    yesterday




    @EmilioPisanty Thank you for clarifying.
    – N. Steinle
    yesterday










    up vote
    5
    down vote














    In other words, is any basis a set of eigenfunctions of some observable?




    IN general, no, since we require that physical observables have certain properties. Mathematically speaking, however, a function can be expressed in any orthogonal basis, but that doesnt mean that the basis is physically meaningful.



    A wavefunction may be written as a linear combination of any well defined basis (that is, it is orthonormal), and of course in principle one can define an operator that corresponds to that basis. However, this operator does not necessarily correspond to a physical observable. For instance, in the example of the harmonic oscillator potential, we may express the wavefunction of the system as a superposition of coherent states, but the operator you'd define from the basis of coherent states (a raising or lowering operator) does not represent a physical observable.



    So, formally speaking, we require that physical observables be Hermitian, so that their corresponding basis is orthonormal over some interval and their eigenvalues are real-valued (so they provide real-valued expectation values). This is an axiom of the theory of quantum mechanics.






    share|cite|improve this answer

























      up vote
      5
      down vote














      In other words, is any basis a set of eigenfunctions of some observable?




      IN general, no, since we require that physical observables have certain properties. Mathematically speaking, however, a function can be expressed in any orthogonal basis, but that doesnt mean that the basis is physically meaningful.



      A wavefunction may be written as a linear combination of any well defined basis (that is, it is orthonormal), and of course in principle one can define an operator that corresponds to that basis. However, this operator does not necessarily correspond to a physical observable. For instance, in the example of the harmonic oscillator potential, we may express the wavefunction of the system as a superposition of coherent states, but the operator you'd define from the basis of coherent states (a raising or lowering operator) does not represent a physical observable.



      So, formally speaking, we require that physical observables be Hermitian, so that their corresponding basis is orthonormal over some interval and their eigenvalues are real-valued (so they provide real-valued expectation values). This is an axiom of the theory of quantum mechanics.






      share|cite|improve this answer























        up vote
        5
        down vote










        up vote
        5
        down vote










        In other words, is any basis a set of eigenfunctions of some observable?




        IN general, no, since we require that physical observables have certain properties. Mathematically speaking, however, a function can be expressed in any orthogonal basis, but that doesnt mean that the basis is physically meaningful.



        A wavefunction may be written as a linear combination of any well defined basis (that is, it is orthonormal), and of course in principle one can define an operator that corresponds to that basis. However, this operator does not necessarily correspond to a physical observable. For instance, in the example of the harmonic oscillator potential, we may express the wavefunction of the system as a superposition of coherent states, but the operator you'd define from the basis of coherent states (a raising or lowering operator) does not represent a physical observable.



        So, formally speaking, we require that physical observables be Hermitian, so that their corresponding basis is orthonormal over some interval and their eigenvalues are real-valued (so they provide real-valued expectation values). This is an axiom of the theory of quantum mechanics.






        share|cite|improve this answer













        In other words, is any basis a set of eigenfunctions of some observable?




        IN general, no, since we require that physical observables have certain properties. Mathematically speaking, however, a function can be expressed in any orthogonal basis, but that doesnt mean that the basis is physically meaningful.



        A wavefunction may be written as a linear combination of any well defined basis (that is, it is orthonormal), and of course in principle one can define an operator that corresponds to that basis. However, this operator does not necessarily correspond to a physical observable. For instance, in the example of the harmonic oscillator potential, we may express the wavefunction of the system as a superposition of coherent states, but the operator you'd define from the basis of coherent states (a raising or lowering operator) does not represent a physical observable.



        So, formally speaking, we require that physical observables be Hermitian, so that their corresponding basis is orthonormal over some interval and their eigenvalues are real-valued (so they provide real-valued expectation values). This is an axiom of the theory of quantum mechanics.







        share|cite|improve this answer












        share|cite|improve this answer



        share|cite|improve this answer










        answered yesterday









        N. Steinle

        897110




        897110






















            up vote
            1
            down vote













            Probably not, but it may depend on what you mean by an observable.



            Any hermitian operator will have a complete basis set of eigenfunctions. Some hermitian operators have been found to be associated with observables. Operators representing observables are required to be hermitian because they must have real expectation values.



            Any linear sum of hermitian operators would also be hermitian, and if made up of observables that quantity could also be observed. I am unsure how to prove that the set of possible complete bases states is larger than the set that you could construct from arbitrary linear sums of existing observables, but you probably can prove that.






            share|cite|improve this answer








            New contributor




            S.D.A. Thomas is a new contributor to this site. Take care in asking for clarification, commenting, and answering.
            Check out our Code of Conduct.














            • 1




              The OP's question is essentially does every basis correspond to an observable. If you say that there exist Hermetian operators with orthonormal eigenbases that don't correspond to observables, then isn't the answer to the question just "no"?
              – Aaron Stevens
              yesterday















            up vote
            1
            down vote













            Probably not, but it may depend on what you mean by an observable.



            Any hermitian operator will have a complete basis set of eigenfunctions. Some hermitian operators have been found to be associated with observables. Operators representing observables are required to be hermitian because they must have real expectation values.



            Any linear sum of hermitian operators would also be hermitian, and if made up of observables that quantity could also be observed. I am unsure how to prove that the set of possible complete bases states is larger than the set that you could construct from arbitrary linear sums of existing observables, but you probably can prove that.






            share|cite|improve this answer








            New contributor




            S.D.A. Thomas is a new contributor to this site. Take care in asking for clarification, commenting, and answering.
            Check out our Code of Conduct.














            • 1




              The OP's question is essentially does every basis correspond to an observable. If you say that there exist Hermetian operators with orthonormal eigenbases that don't correspond to observables, then isn't the answer to the question just "no"?
              – Aaron Stevens
              yesterday













            up vote
            1
            down vote










            up vote
            1
            down vote









            Probably not, but it may depend on what you mean by an observable.



            Any hermitian operator will have a complete basis set of eigenfunctions. Some hermitian operators have been found to be associated with observables. Operators representing observables are required to be hermitian because they must have real expectation values.



            Any linear sum of hermitian operators would also be hermitian, and if made up of observables that quantity could also be observed. I am unsure how to prove that the set of possible complete bases states is larger than the set that you could construct from arbitrary linear sums of existing observables, but you probably can prove that.






            share|cite|improve this answer








            New contributor




            S.D.A. Thomas is a new contributor to this site. Take care in asking for clarification, commenting, and answering.
            Check out our Code of Conduct.









            Probably not, but it may depend on what you mean by an observable.



            Any hermitian operator will have a complete basis set of eigenfunctions. Some hermitian operators have been found to be associated with observables. Operators representing observables are required to be hermitian because they must have real expectation values.



            Any linear sum of hermitian operators would also be hermitian, and if made up of observables that quantity could also be observed. I am unsure how to prove that the set of possible complete bases states is larger than the set that you could construct from arbitrary linear sums of existing observables, but you probably can prove that.







            share|cite|improve this answer








            New contributor




            S.D.A. Thomas is a new contributor to this site. Take care in asking for clarification, commenting, and answering.
            Check out our Code of Conduct.









            share|cite|improve this answer



            share|cite|improve this answer






            New contributor




            S.D.A. Thomas is a new contributor to this site. Take care in asking for clarification, commenting, and answering.
            Check out our Code of Conduct.









            answered yesterday









            S.D.A. Thomas

            112




            112




            New contributor




            S.D.A. Thomas is a new contributor to this site. Take care in asking for clarification, commenting, and answering.
            Check out our Code of Conduct.





            New contributor





            S.D.A. Thomas is a new contributor to this site. Take care in asking for clarification, commenting, and answering.
            Check out our Code of Conduct.






            S.D.A. Thomas is a new contributor to this site. Take care in asking for clarification, commenting, and answering.
            Check out our Code of Conduct.








            • 1




              The OP's question is essentially does every basis correspond to an observable. If you say that there exist Hermetian operators with orthonormal eigenbases that don't correspond to observables, then isn't the answer to the question just "no"?
              – Aaron Stevens
              yesterday














            • 1




              The OP's question is essentially does every basis correspond to an observable. If you say that there exist Hermetian operators with orthonormal eigenbases that don't correspond to observables, then isn't the answer to the question just "no"?
              – Aaron Stevens
              yesterday








            1




            1




            The OP's question is essentially does every basis correspond to an observable. If you say that there exist Hermetian operators with orthonormal eigenbases that don't correspond to observables, then isn't the answer to the question just "no"?
            – Aaron Stevens
            yesterday




            The OP's question is essentially does every basis correspond to an observable. If you say that there exist Hermetian operators with orthonormal eigenbases that don't correspond to observables, then isn't the answer to the question just "no"?
            – Aaron Stevens
            yesterday


















             

            draft saved


            draft discarded



















































             


            draft saved


            draft discarded














            StackExchange.ready(
            function () {
            StackExchange.openid.initPostLogin('.new-post-login', 'https%3a%2f%2fphysics.stackexchange.com%2fquestions%2f442583%2fdoes-any-basis-belong-to-a-set-of-eigenfunctions-of-some-observable%23new-answer', 'question_page');
            }
            );

            Post as a guest















            Required, but never shown





















































            Required, but never shown














            Required, but never shown












            Required, but never shown







            Required, but never shown

































            Required, but never shown














            Required, but never shown












            Required, but never shown







            Required, but never shown







            Popular posts from this blog

            サソリ

            広島県道265号伴広島線

            Accessing regular linux commands in Huawei's Dopra Linux